Ensino SuperiorCálculo de Derivadas Tópico resolvido

Poste aqui problemas sobre assuntos estudados no Ensino Superior (exceto os cobrados em concursos públicos e escolas militares).

Moderador: [ Moderadores TTB ]

Avatar do usuário
Autor do Tópico
marcos5566
Junior
Mensagens: 18
Registrado em: Qui 22 Fev, 2018 11:11
Última visita: 20-04-18
Fev 2018 22 11:30

Cálculo de Derivadas

Mensagem não lida por marcos5566 »

Uma empresa fabricante de caixas d’água deseja lançar um novo tanque em formato cilíndrico no mercado. Então pediu-se à equipe de desenvolvimento que preparasse uma proposta de projeto com capacidade de 1000L. Como a equipe pode determinar a medida do raio da base e da altura do reservatório de modo que a quantidade de material utilizada para sua fabricação seja mínima?

Preciso da resposta urgente, por favor me ajudem.




Avatar do usuário
lorramrj
3 - Destaque
Mensagens: 372
Registrado em: Qui 27 Nov, 2014 15:46
Última visita: 28-02-24
Fev 2018 22 13:10

Re: Cálculo de Derivadas

Mensagem não lida por lorramrj »

EDIT:

Temos o cilíndro:
cilindro.jpg
cilindro.jpg (3.94 KiB) Exibido 1506 vezes
Volume do cilíndro é dado por: [tex3]V = A_{base} \cdot h = \pi\cdot r^2\cdot h[/tex3]

Temos a restrição que: [tex3]V=1000L=1m^3\rightarrow \boxed{ h = \dfrac{1}{\pi\cdot r^2}} [/tex3]

Área total: [tex3]A_t = A_{lateral} + 2\cdot A_{base} \\= 2\cdot \pi\cdot r\cdot h + 2\cdot \pi\cdot r^2= 2\cdot \pi\cdot r\cdot \dfrac{1}{\pi\cdot r^2} + 2\cdot \pi\cdot r^2 = \dfrac{2}{r} + 2\cdot \pi\cdot r^2 = \dfrac{2+2\cdot \pi\cdot r^3}{r}[/tex3]

Temos uma função da área total dependendo do raio. Vamos minimizar essa função, derivando [tex3]A_t [/tex3] em relação a r:

Regra do Quociente:
[tex3]A_{t}'(r) = \dfrac{(6\cdot \pi\cdot r^2)\cdot r - (2+2\cdot \pi\cdot r^3)\cdot (1)}{r^2} = \dfrac{6\pi r^3 - 2\pi r^3 - 2}{r^2} =\dfrac{4\pi r^3-2}{r^2} [/tex3]

Procurando o ponto crítico: [tex3]A'_t(r)=0[/tex3] , como o denominador é sempre positivo e diferente de zero, pois [tex3]r>0[/tex3] .
Estudamos o sinal do numerador:

[tex3]4\pi r^3-2= 0 \rightarrow \boxed {r = \sqrt[3]{\dfrac{1}{2\pi}}m}[/tex3]

Logo, para [tex3]h\(\sqrt[3]{\dfrac{1}{2\pi}}m\) = \dfrac{1}{\pi\cdot \[\(\dfrac{1}{2\pi}\)^{\dfrac{1}{3}}\]^2}= \dfrac{1}{\pi}\cdot \sqrt[3]{\dfrac{1}{\(\dfrac{1}{4\pi^2}\)}}\rightarrow \boxed {h=\dfrac{\sqrt[3]{4\pi^2}}{\pi} m} [/tex3] .

Testando:
[tex3]V = \pi\cdot r^2\cdot h = \pi \cdot \( \sqrt[3]{\dfrac{1}{2\pi}}\)^2\cdot \(\dfrac{\sqrt[3]{4\pi^2}}{\pi}\) =1,0m^3[/tex3] ...OK

Última edição: lorramrj (Qui 22 Fev, 2018 14:37). Total de 4 vezes.


Engenharia da Computação | PUC-RIO

O que sabemos não é muito. O que não sabemos é imenso.”
:-> [tex3]\textbf{S. P. Laplace}[/tex3]

Avatar do usuário
jvmago
5 - Mestre
Mensagens: 2713
Registrado em: Qui 06 Jul, 2017 14:54
Última visita: 24-02-24
Fev 2018 22 13:35

Re: Cálculo de Derivadas

Mensagem não lida por jvmago »

O certo não seria [tex3]A_{t}'(r) = \dfrac{(6.\pi.r^2).r - (2+2.\pi.r^3).(1)}{r^2}[/tex3] ?


Não importa se você é magrinho ou gordinho, alto ou baixo, o que te difere dos outros é quando expõe seus conhecimentos.

Avatar do usuário
lorramrj
3 - Destaque
Mensagens: 372
Registrado em: Qui 27 Nov, 2014 15:46
Última visita: 28-02-24
Fev 2018 22 13:46

Re: Cálculo de Derivadas

Mensagem não lida por lorramrj »

jvmago escreveu:
Qui 22 Fev, 2018 13:35
O certo não seria [tex3]A_{t}'(r) = \dfrac{(6.\pi.r^2).r - (2+2.\pi.r^3).(1)}{r^2}[/tex3] ?
Sim, passou batido valeu!


Engenharia da Computação | PUC-RIO

O que sabemos não é muito. O que não sabemos é imenso.”
:-> [tex3]\textbf{S. P. Laplace}[/tex3]

Avatar do usuário
jvmago
5 - Mestre
Mensagens: 2713
Registrado em: Qui 06 Jul, 2017 14:54
Última visita: 24-02-24
Fev 2018 22 13:51

Re: Cálculo de Derivadas

Mensagem não lida por jvmago »

Da dedução do Lorran, [tex3]S_{t}=\dfrac{2(1+.\pi.r^3)}{r}[/tex3]
[tex3]S'_{t}=\frac{2(2\pi r^3-1)}{r^2}[/tex3]
calculando o [tex3]S'_{t}=0\rightarrow r^3=\frac{1}{2\pi }\rightarrow r=\frac{\sqrt[3]{4\pi ^2}}{2\pi }[/tex3] [tex3]r\neq 0[/tex3]
substituindo na formula da altura:
[tex3]H=\frac{1}{\pi r^2}\rightarrow H=\frac{\sqrt[3]{4\pi ^2}}{\pi }[/tex3]
Última edição: jvmago (Qui 22 Fev, 2018 13:52). Total de 1 vez.


Não importa se você é magrinho ou gordinho, alto ou baixo, o que te difere dos outros é quando expõe seus conhecimentos.

Avatar do usuário
Autor do Tópico
marcos5566
Junior
Mensagens: 18
Registrado em: Qui 22 Fev, 2018 11:11
Última visita: 20-04-18
Fev 2018 22 14:06

Re: Cálculo de Derivadas

Mensagem não lida por marcos5566 »

jvmago escreveu:
Qui 22 Fev, 2018 13:51
Da dedução do Lorran, [tex3]S_{t}=\dfrac{2(1+.\pi.r^3)}{r}[/tex3]
[tex3]S'_{t}=\frac{2(2\pi r^3-1)}{r^2}[/tex3]
calculando o [tex3]S'_{t}=0\rightarrow r^3=\frac{1}{2\pi }\rightarrow r=\frac{\sqrt[3]{4\pi ^2}}{2\pi }[/tex3] [tex3]r\neq 0[/tex3]
substituindo na formula da altura:
[tex3]H=\frac{1}{\pi r^2}\rightarrow H=\frac{\sqrt[3]{4\pi ^2}}{\pi }[/tex3]
Essa seria a resposta correta então????



Avatar do usuário
jvmago
5 - Mestre
Mensagens: 2713
Registrado em: Qui 06 Jul, 2017 14:54
Última visita: 24-02-24
Fev 2018 22 14:09

Re: Cálculo de Derivadas

Mensagem não lida por jvmago »

marcos5566 escreveu:
Qui 22 Fev, 2018 14:06
jvmago escreveu:
Qui 22 Fev, 2018 13:51
Da dedução do Lorran, [tex3]S_{t}=\dfrac{2(1+.\pi.r^3)}{r}[/tex3]
[tex3]S'_{t}=\frac{2(2\pi r^3-1)}{r^2}[/tex3]
calculando o [tex3]S'_{t}=0\rightarrow r^3=\frac{1}{2\pi }\rightarrow r=\frac{\sqrt[3]{4\pi ^2}}{2\pi }[/tex3] [tex3]r\neq 0[/tex3]
substituindo na formula da altura:
[tex3]H=\frac{1}{\pi r^2}\rightarrow H=\frac{\sqrt[3]{4\pi ^2}}{\pi }[/tex3]
Essa seria a resposta correta então????
Se eu não tiver me equivacado, sim


Não importa se você é magrinho ou gordinho, alto ou baixo, o que te difere dos outros é quando expõe seus conhecimentos.

Avatar do usuário
Autor do Tópico
marcos5566
Junior
Mensagens: 18
Registrado em: Qui 22 Fev, 2018 11:11
Última visita: 20-04-18
Fev 2018 22 14:40

Re: Cálculo de Derivadas

Mensagem não lida por marcos5566 »

jvmago escreveu:
Qui 22 Fev, 2018 14:09
marcos5566 escreveu:
Qui 22 Fev, 2018 14:06
jvmago escreveu:
Qui 22 Fev, 2018 13:51
Da dedução do Lorran, [tex3]S_{t}=\dfrac{2(1+.\pi.r^3)}{r}[/tex3]
[tex3]S'_{t}=\frac{2(2\pi r^3-1)}{r^2}[/tex3]
calculando o [tex3]S'_{t}=0\rightarrow r^3=\frac{1}{2\pi }\rightarrow r=\frac{\sqrt[3]{4\pi ^2}}{2\pi }[/tex3] [tex3]r\neq 0[/tex3]
substituindo na formula da altura:
[tex3]H=\frac{1}{\pi r^2}\rightarrow H=\frac{\sqrt[3]{4\pi ^2}}{\pi }[/tex3]
Essa seria a resposta correta então????
Se eu não tiver me equivacado, sim
Beleza, Valeu !!!




Responder
  • Tópicos Semelhantes
    Respostas
    Exibições
    Última msg

Voltar para “Ensino Superior”